Difference between revisions of "Antiderivative of arctan"

From specialfunctionswiki
Jump to: navigation, search
(Created page with "<div class="toccolours mw-collapsible mw-collapsed"> <strong>Theorem:</strong> The following formula holds: $$\displaystyle\int \mathrm{arctan}(z)...")
(No difference)

Revision as of 04:15, 16 May 2016

Theorem: The following formula holds: $$\displaystyle\int \mathrm{arctan}(z) = z\mathrm{arctan}(z) - \dfrac{1}{2}\log(1+z^2)+C,$$ where $\mathrm{arctan}$ denotes the inverse tangent and $\log$ denotes the logarithm.

Proof: